Find the conditional probability density function of $Y$ given $X=x$Probability Density Function - GammaFinding quantiles based on probability density functionsProbability density function of random variable $X-Y$Find the conditional probability of a probability density functionConditional Distribution from Conditional densityFind the marginal and conditional densities without explicitly having the joint density?Find the conditional probability density function of $X$ given $Y=y$Computing probability function of $Y$ in terms of $f_X$How can I get the probability density function?Compute the probability density function of Y

How to test the sharpness of a knife?

How many people need to be born every 8 years to sustain population?

If Captain Marvel (MCU) were to have a child with a human male, would the child be human or Kree?

El Dorado Word Puzzle II: Videogame Edition

Why do Radio Buttons not fill the entire outer circle?

Why would five hundred and five be same as one?

Does Doodling or Improvising on the Piano Have Any Benefits?

How to preserve electronics (computers, iPads and phones) for hundreds of years

How much do grades matter for a future academia position?

Can I cause damage to electrical appliances by unplugging them when they are turned on?

Why is the Sun approximated as a black body at ~ 5800 K?

Why the "ls" command is showing the permissions of files in a FAT32 partition?

Do I have to take mana from my deck or hand when tapping a dual land?

Pre-Employment Background Check With Consent For Future Checks

Is there anyway, I can have two passwords for my wi-fi

Air travel with refrigerated insulin

How to write Quadratic equation with negative coefficient

PTIJ: Which Dr. Seuss books should one obtain?

Sigmoid with a slope but no asymptotes?

Deciphering cause of death?

Why does a 97 / 92 key piano exist by Bösendorfer?

What the heck is gets(stdin) on site coderbyte?

Would this string work as string?

Has the laser at Magurele, Romania reached a tenth of the Sun's power?



Find the conditional probability density function of $Y$ given $X=x$


Probability Density Function - GammaFinding quantiles based on probability density functionsProbability density function of random variable $X-Y$Find the conditional probability of a probability density functionConditional Distribution from Conditional densityFind the marginal and conditional densities without explicitly having the joint density?Find the conditional probability density function of $X$ given $Y=y$Computing probability function of $Y$ in terms of $f_X$How can I get the probability density function?Compute the probability density function of Y













0












$begingroup$


Suppose $Y$ is a continuous random variable with probability density function $f(y)=192over y^4$, for $ygeq 4$ ($0$ otherwise). If the conditional distribution of X given $Y = y$ is a uniform distribution on $[0, y]$. Find the conditional probability density function of $Y$ given $X = x$.



My clue is that:
$$f_Y(x|y)=f_X,Y(x,y)overf_Y(y)=1over y$$
then we get $$f_X,Y(x,y)=192over y^5$$
$$f_Y(y|x)=f_X,Y(x,y)overf_X(x)$$
But I don't know how to find the domain of $Y$ in terms of $X$ to get $f_X(x)$.

Am I on the right track?










share|cite|improve this question











$endgroup$
















    0












    $begingroup$


    Suppose $Y$ is a continuous random variable with probability density function $f(y)=192over y^4$, for $ygeq 4$ ($0$ otherwise). If the conditional distribution of X given $Y = y$ is a uniform distribution on $[0, y]$. Find the conditional probability density function of $Y$ given $X = x$.



    My clue is that:
    $$f_Y(x|y)=f_X,Y(x,y)overf_Y(y)=1over y$$
    then we get $$f_X,Y(x,y)=192over y^5$$
    $$f_Y(y|x)=f_X,Y(x,y)overf_X(x)$$
    But I don't know how to find the domain of $Y$ in terms of $X$ to get $f_X(x)$.

    Am I on the right track?










    share|cite|improve this question











    $endgroup$














      0












      0








      0





      $begingroup$


      Suppose $Y$ is a continuous random variable with probability density function $f(y)=192over y^4$, for $ygeq 4$ ($0$ otherwise). If the conditional distribution of X given $Y = y$ is a uniform distribution on $[0, y]$. Find the conditional probability density function of $Y$ given $X = x$.



      My clue is that:
      $$f_Y(x|y)=f_X,Y(x,y)overf_Y(y)=1over y$$
      then we get $$f_X,Y(x,y)=192over y^5$$
      $$f_Y(y|x)=f_X,Y(x,y)overf_X(x)$$
      But I don't know how to find the domain of $Y$ in terms of $X$ to get $f_X(x)$.

      Am I on the right track?










      share|cite|improve this question











      $endgroup$




      Suppose $Y$ is a continuous random variable with probability density function $f(y)=192over y^4$, for $ygeq 4$ ($0$ otherwise). If the conditional distribution of X given $Y = y$ is a uniform distribution on $[0, y]$. Find the conditional probability density function of $Y$ given $X = x$.



      My clue is that:
      $$f_Y(x|y)=f_X,Y(x,y)overf_Y(y)=1over y$$
      then we get $$f_X,Y(x,y)=192over y^5$$
      $$f_Y(y|x)=f_X,Y(x,y)overf_X(x)$$
      But I don't know how to find the domain of $Y$ in terms of $X$ to get $f_X(x)$.

      Am I on the right track?







      probability






      share|cite|improve this question















      share|cite|improve this question













      share|cite|improve this question




      share|cite|improve this question








      edited Mar 14 at 6:15









      Rócherz

      2,9863821




      2,9863821










      asked Oct 1 '18 at 18:06









      Yibei HeYibei He

      3139




      3139




















          1 Answer
          1






          active

          oldest

          votes


















          1












          $begingroup$

          Note that the expression for the joint density $f_X,Y(x,y)$ that you computed only holds for $x$ and $y$ satisfying $x ge 0$ and $y ge maxx, 4$; it is zero otherwise. Thus for $x ge 0$ we have
          $$f_X(x) = int_-infty^infty f_X,Y(x,z) , dz = int_maxx, 4^infty frac192z^5 , dz.$$




          Continuing the computation leads to
          $$f_X(x) = 48 maxx, 4^-4 = begincases48 / 4^4 & x in [0, 4] \ 48 / x^4 & x > 4endcases$$
          So,
          $$int_0^infty f_X(x) , dx = 4 cdot frac484^4 + int_4^infty frac48x^4 , dx
          = frac34 + frac164^3 = 1.$$






          share|cite|improve this answer











          $endgroup$












          • $begingroup$
            So when you integral the joint pdf from x to ∞, you assumed that x is greater than 4, right? Otherwise it will be from 4 to ∞. But does it contradict with 0≤x?
            $endgroup$
            – Yibei He
            Oct 1 '18 at 20:13










          • $begingroup$
            @YibeiHe Good catch; I've edited my answer.
            $endgroup$
            – angryavian
            Oct 1 '18 at 22:12










          • $begingroup$
            I'm still quite confused about the domain. Do you mean $f_X(x)$ will have two case, one is x=[0,4] and the other is x=[4,∞]? If I get $f_X(x)$ in this way, when I integral $f_X(x)$ in both case, I can't get the summation to 1. How does that happen?
            $endgroup$
            – Yibei He
            Oct 2 '18 at 2:44










          • $begingroup$
            @YibeiHe See my edit.
            $endgroup$
            – angryavian
            Oct 2 '18 at 2:51










          Your Answer





          StackExchange.ifUsing("editor", function ()
          return StackExchange.using("mathjaxEditing", function ()
          StackExchange.MarkdownEditor.creationCallbacks.add(function (editor, postfix)
          StackExchange.mathjaxEditing.prepareWmdForMathJax(editor, postfix, [["$", "$"], ["\\(","\\)"]]);
          );
          );
          , "mathjax-editing");

          StackExchange.ready(function()
          var channelOptions =
          tags: "".split(" "),
          id: "69"
          ;
          initTagRenderer("".split(" "), "".split(" "), channelOptions);

          StackExchange.using("externalEditor", function()
          // Have to fire editor after snippets, if snippets enabled
          if (StackExchange.settings.snippets.snippetsEnabled)
          StackExchange.using("snippets", function()
          createEditor();
          );

          else
          createEditor();

          );

          function createEditor()
          StackExchange.prepareEditor(
          heartbeatType: 'answer',
          autoActivateHeartbeat: false,
          convertImagesToLinks: true,
          noModals: true,
          showLowRepImageUploadWarning: true,
          reputationToPostImages: 10,
          bindNavPrevention: true,
          postfix: "",
          imageUploader:
          brandingHtml: "Powered by u003ca class="icon-imgur-white" href="https://imgur.com/"u003eu003c/au003e",
          contentPolicyHtml: "User contributions licensed under u003ca href="https://creativecommons.org/licenses/by-sa/3.0/"u003ecc by-sa 3.0 with attribution requiredu003c/au003e u003ca href="https://stackoverflow.com/legal/content-policy"u003e(content policy)u003c/au003e",
          allowUrls: true
          ,
          noCode: true, onDemand: true,
          discardSelector: ".discard-answer"
          ,immediatelyShowMarkdownHelp:true
          );



          );













          draft saved

          draft discarded


















          StackExchange.ready(
          function ()
          StackExchange.openid.initPostLogin('.new-post-login', 'https%3a%2f%2fmath.stackexchange.com%2fquestions%2f2938286%2ffind-the-conditional-probability-density-function-of-y-given-x-x%23new-answer', 'question_page');

          );

          Post as a guest















          Required, but never shown

























          1 Answer
          1






          active

          oldest

          votes








          1 Answer
          1






          active

          oldest

          votes









          active

          oldest

          votes






          active

          oldest

          votes









          1












          $begingroup$

          Note that the expression for the joint density $f_X,Y(x,y)$ that you computed only holds for $x$ and $y$ satisfying $x ge 0$ and $y ge maxx, 4$; it is zero otherwise. Thus for $x ge 0$ we have
          $$f_X(x) = int_-infty^infty f_X,Y(x,z) , dz = int_maxx, 4^infty frac192z^5 , dz.$$




          Continuing the computation leads to
          $$f_X(x) = 48 maxx, 4^-4 = begincases48 / 4^4 & x in [0, 4] \ 48 / x^4 & x > 4endcases$$
          So,
          $$int_0^infty f_X(x) , dx = 4 cdot frac484^4 + int_4^infty frac48x^4 , dx
          = frac34 + frac164^3 = 1.$$






          share|cite|improve this answer











          $endgroup$












          • $begingroup$
            So when you integral the joint pdf from x to ∞, you assumed that x is greater than 4, right? Otherwise it will be from 4 to ∞. But does it contradict with 0≤x?
            $endgroup$
            – Yibei He
            Oct 1 '18 at 20:13










          • $begingroup$
            @YibeiHe Good catch; I've edited my answer.
            $endgroup$
            – angryavian
            Oct 1 '18 at 22:12










          • $begingroup$
            I'm still quite confused about the domain. Do you mean $f_X(x)$ will have two case, one is x=[0,4] and the other is x=[4,∞]? If I get $f_X(x)$ in this way, when I integral $f_X(x)$ in both case, I can't get the summation to 1. How does that happen?
            $endgroup$
            – Yibei He
            Oct 2 '18 at 2:44










          • $begingroup$
            @YibeiHe See my edit.
            $endgroup$
            – angryavian
            Oct 2 '18 at 2:51















          1












          $begingroup$

          Note that the expression for the joint density $f_X,Y(x,y)$ that you computed only holds for $x$ and $y$ satisfying $x ge 0$ and $y ge maxx, 4$; it is zero otherwise. Thus for $x ge 0$ we have
          $$f_X(x) = int_-infty^infty f_X,Y(x,z) , dz = int_maxx, 4^infty frac192z^5 , dz.$$




          Continuing the computation leads to
          $$f_X(x) = 48 maxx, 4^-4 = begincases48 / 4^4 & x in [0, 4] \ 48 / x^4 & x > 4endcases$$
          So,
          $$int_0^infty f_X(x) , dx = 4 cdot frac484^4 + int_4^infty frac48x^4 , dx
          = frac34 + frac164^3 = 1.$$






          share|cite|improve this answer











          $endgroup$












          • $begingroup$
            So when you integral the joint pdf from x to ∞, you assumed that x is greater than 4, right? Otherwise it will be from 4 to ∞. But does it contradict with 0≤x?
            $endgroup$
            – Yibei He
            Oct 1 '18 at 20:13










          • $begingroup$
            @YibeiHe Good catch; I've edited my answer.
            $endgroup$
            – angryavian
            Oct 1 '18 at 22:12










          • $begingroup$
            I'm still quite confused about the domain. Do you mean $f_X(x)$ will have two case, one is x=[0,4] and the other is x=[4,∞]? If I get $f_X(x)$ in this way, when I integral $f_X(x)$ in both case, I can't get the summation to 1. How does that happen?
            $endgroup$
            – Yibei He
            Oct 2 '18 at 2:44










          • $begingroup$
            @YibeiHe See my edit.
            $endgroup$
            – angryavian
            Oct 2 '18 at 2:51













          1












          1








          1





          $begingroup$

          Note that the expression for the joint density $f_X,Y(x,y)$ that you computed only holds for $x$ and $y$ satisfying $x ge 0$ and $y ge maxx, 4$; it is zero otherwise. Thus for $x ge 0$ we have
          $$f_X(x) = int_-infty^infty f_X,Y(x,z) , dz = int_maxx, 4^infty frac192z^5 , dz.$$




          Continuing the computation leads to
          $$f_X(x) = 48 maxx, 4^-4 = begincases48 / 4^4 & x in [0, 4] \ 48 / x^4 & x > 4endcases$$
          So,
          $$int_0^infty f_X(x) , dx = 4 cdot frac484^4 + int_4^infty frac48x^4 , dx
          = frac34 + frac164^3 = 1.$$






          share|cite|improve this answer











          $endgroup$



          Note that the expression for the joint density $f_X,Y(x,y)$ that you computed only holds for $x$ and $y$ satisfying $x ge 0$ and $y ge maxx, 4$; it is zero otherwise. Thus for $x ge 0$ we have
          $$f_X(x) = int_-infty^infty f_X,Y(x,z) , dz = int_maxx, 4^infty frac192z^5 , dz.$$




          Continuing the computation leads to
          $$f_X(x) = 48 maxx, 4^-4 = begincases48 / 4^4 & x in [0, 4] \ 48 / x^4 & x > 4endcases$$
          So,
          $$int_0^infty f_X(x) , dx = 4 cdot frac484^4 + int_4^infty frac48x^4 , dx
          = frac34 + frac164^3 = 1.$$







          share|cite|improve this answer














          share|cite|improve this answer



          share|cite|improve this answer








          edited Oct 2 '18 at 2:51

























          answered Oct 1 '18 at 18:10









          angryavianangryavian

          42.2k23481




          42.2k23481











          • $begingroup$
            So when you integral the joint pdf from x to ∞, you assumed that x is greater than 4, right? Otherwise it will be from 4 to ∞. But does it contradict with 0≤x?
            $endgroup$
            – Yibei He
            Oct 1 '18 at 20:13










          • $begingroup$
            @YibeiHe Good catch; I've edited my answer.
            $endgroup$
            – angryavian
            Oct 1 '18 at 22:12










          • $begingroup$
            I'm still quite confused about the domain. Do you mean $f_X(x)$ will have two case, one is x=[0,4] and the other is x=[4,∞]? If I get $f_X(x)$ in this way, when I integral $f_X(x)$ in both case, I can't get the summation to 1. How does that happen?
            $endgroup$
            – Yibei He
            Oct 2 '18 at 2:44










          • $begingroup$
            @YibeiHe See my edit.
            $endgroup$
            – angryavian
            Oct 2 '18 at 2:51
















          • $begingroup$
            So when you integral the joint pdf from x to ∞, you assumed that x is greater than 4, right? Otherwise it will be from 4 to ∞. But does it contradict with 0≤x?
            $endgroup$
            – Yibei He
            Oct 1 '18 at 20:13










          • $begingroup$
            @YibeiHe Good catch; I've edited my answer.
            $endgroup$
            – angryavian
            Oct 1 '18 at 22:12










          • $begingroup$
            I'm still quite confused about the domain. Do you mean $f_X(x)$ will have two case, one is x=[0,4] and the other is x=[4,∞]? If I get $f_X(x)$ in this way, when I integral $f_X(x)$ in both case, I can't get the summation to 1. How does that happen?
            $endgroup$
            – Yibei He
            Oct 2 '18 at 2:44










          • $begingroup$
            @YibeiHe See my edit.
            $endgroup$
            – angryavian
            Oct 2 '18 at 2:51















          $begingroup$
          So when you integral the joint pdf from x to ∞, you assumed that x is greater than 4, right? Otherwise it will be from 4 to ∞. But does it contradict with 0≤x?
          $endgroup$
          – Yibei He
          Oct 1 '18 at 20:13




          $begingroup$
          So when you integral the joint pdf from x to ∞, you assumed that x is greater than 4, right? Otherwise it will be from 4 to ∞. But does it contradict with 0≤x?
          $endgroup$
          – Yibei He
          Oct 1 '18 at 20:13












          $begingroup$
          @YibeiHe Good catch; I've edited my answer.
          $endgroup$
          – angryavian
          Oct 1 '18 at 22:12




          $begingroup$
          @YibeiHe Good catch; I've edited my answer.
          $endgroup$
          – angryavian
          Oct 1 '18 at 22:12












          $begingroup$
          I'm still quite confused about the domain. Do you mean $f_X(x)$ will have two case, one is x=[0,4] and the other is x=[4,∞]? If I get $f_X(x)$ in this way, when I integral $f_X(x)$ in both case, I can't get the summation to 1. How does that happen?
          $endgroup$
          – Yibei He
          Oct 2 '18 at 2:44




          $begingroup$
          I'm still quite confused about the domain. Do you mean $f_X(x)$ will have two case, one is x=[0,4] and the other is x=[4,∞]? If I get $f_X(x)$ in this way, when I integral $f_X(x)$ in both case, I can't get the summation to 1. How does that happen?
          $endgroup$
          – Yibei He
          Oct 2 '18 at 2:44












          $begingroup$
          @YibeiHe See my edit.
          $endgroup$
          – angryavian
          Oct 2 '18 at 2:51




          $begingroup$
          @YibeiHe See my edit.
          $endgroup$
          – angryavian
          Oct 2 '18 at 2:51

















          draft saved

          draft discarded
















































          Thanks for contributing an answer to Mathematics Stack Exchange!


          • Please be sure to answer the question. Provide details and share your research!

          But avoid


          • Asking for help, clarification, or responding to other answers.

          • Making statements based on opinion; back them up with references or personal experience.

          Use MathJax to format equations. MathJax reference.


          To learn more, see our tips on writing great answers.




          draft saved


          draft discarded














          StackExchange.ready(
          function ()
          StackExchange.openid.initPostLogin('.new-post-login', 'https%3a%2f%2fmath.stackexchange.com%2fquestions%2f2938286%2ffind-the-conditional-probability-density-function-of-y-given-x-x%23new-answer', 'question_page');

          );

          Post as a guest















          Required, but never shown





















































          Required, but never shown














          Required, but never shown












          Required, but never shown







          Required, but never shown

































          Required, but never shown














          Required, but never shown












          Required, but never shown







          Required, but never shown







          Popular posts from this blog

          Lowndes Grove History Architecture References Navigation menu32°48′6″N 79°57′58″W / 32.80167°N 79.96611°W / 32.80167; -79.9661132°48′6″N 79°57′58″W / 32.80167°N 79.96611°W / 32.80167; -79.9661178002500"National Register Information System"Historic houses of South Carolina"Lowndes Grove""+32° 48' 6.00", −79° 57' 58.00""Lowndes Grove, Charleston County (260 St. Margaret St., Charleston)""Lowndes Grove"The Charleston ExpositionIt Happened in South Carolina"Lowndes Grove (House), Saint Margaret Street & Sixth Avenue, Charleston, Charleston County, SC(Photographs)"Plantations of the Carolina Low Countrye

          random experiment with two different functions on unit interval Announcing the arrival of Valued Associate #679: Cesar Manara Planned maintenance scheduled April 23, 2019 at 00:00UTC (8:00pm US/Eastern)Random variable and probability space notionsRandom Walk with EdgesFinding functions where the increase over a random interval is Poisson distributedNumber of days until dayCan an observed event in fact be of zero probability?Unit random processmodels of coins and uniform distributionHow to get the number of successes given $n$ trials , probability $P$ and a random variable $X$Absorbing Markov chain in a computer. Is “almost every” turned into always convergence in computer executions?Stopped random walk is not uniformly integrable

          How should I support this large drywall patch? Planned maintenance scheduled April 23, 2019 at 00:00UTC (8:00pm US/Eastern) Announcing the arrival of Valued Associate #679: Cesar Manara Unicorn Meta Zoo #1: Why another podcast?How do I cover large gaps in drywall?How do I keep drywall around a patch from crumbling?Can I glue a second layer of drywall?How to patch long strip on drywall?Large drywall patch: how to avoid bulging seams?Drywall Mesh Patch vs. Bulge? To remove or not to remove?How to fix this drywall job?Prep drywall before backsplashWhat's the best way to fix this horrible drywall patch job?Drywall patching using 3M Patch Plus Primer